Difference between revisions of "2020 AMC 10A Problems/Problem 22"

(Solution 1)
(Solution 1)
Line 6: Line 6:
  
 
== Solution 1 ==
 
== Solution 1 ==
Let <math>a = \left\lfloor \frac{998}n \right\rfloor</math>. If the expression <math>n \le 1000</math> is<cmath>\left\lfloor \dfrac{998}{n} \right\rfloor+\left\lfloor \dfrac{999}{n} \right\rfloor+\left\lfloor \dfrac{1000}{n}\right \rfloor</cmath>is not divisible by <math>3</math>, then the three terms in the expression must be <math>(a, a + 1, a + 1)</math>, which would imply that <math>n</math> is a divisor of <math>999</math> but not <math>1000</math>, or <math>(a, a, a + 1)</math>, which would imply that <math>n</math> is a divisor of <math>1000</math> but not <math>999</math>. <math>999 = 3^3 \cdot 37</math> has <math>4 \cdot 2 = 8</math> factors, and <math>1000 = 2^3 \cdot 5^3</math> has <math>4 \cdot 4 = 16</math> factors. However, <math>n = 1</math> does not work because <math>1</math> a divisor of both <math>999</math> and <math>1000</math>, and since <math>1</math> is counted twice, the answer is <math>16 + 8 - 2 = \boxed{\textbf{(A) }22}</math>.
+
Let <math>a = \left\lfloor \frac{998}n \right\rfloor</math>. If the expression <math>\left\lfloor \dfrac{998}{n} \right\rfloor+\left\lfloor \dfrac{999}{n} \right\rfloor+\left\lfloor \dfrac{1000}{n}\right \rfloor</math><math>is not divisible by </math>3<math>, then the three terms in the expression must be </math>(a, a + 1, a + 1)<math>, which would imply that </math>n<math> is a divisor of </math>999<math> but not </math>1000<math>, or </math>(a, a, a + 1)<math>, which would imply that </math>n<math> is a divisor of </math>1000<math> but not </math>999<math>. </math>999 = 3^3 \cdot 37<math> has </math>4 \cdot 2 = 8<math> factors, and </math>1000 = 2^3 \cdot 5^3<math> has </math>4 \cdot 4 = 16<math> factors. However, </math>n = 1<math> does not work because </math>1<math> a divisor of both </math>999<math> and </math>1000<math>, and since </math>1<math> is counted twice, the answer is </math>16 + 8 - 2 = \boxed{\textbf{(A) }22}$.
  
 
== Solution 2 ==
 
== Solution 2 ==

Revision as of 19:25, 1 February 2020

Problem

For how many positive integers $n \le 1000$ is\[\left\lfloor \dfrac{998}{n} \right\rfloor+\left\lfloor \dfrac{999}{n} \right\rfloor+\left\lfloor \dfrac{1000}{n}\right \rfloor\]not divisible by $3$? (Recall that $\lfloor x \rfloor$ is the greatest integer less than or equal to $x$.)

$\textbf{(A) } 22 \qquad\textbf{(B) } 23 \qquad\textbf{(C) } 24 \qquad\textbf{(D) } 25 \qquad\textbf{(E) } 26$

Solution 1

Let $a = \left\lfloor \frac{998}n \right\rfloor$. If the expression $\left\lfloor \dfrac{998}{n} \right\rfloor+\left\lfloor \dfrac{999}{n} \right\rfloor+\left\lfloor \dfrac{1000}{n}\right \rfloor$$is not divisible by$3$, then the three terms in the expression must be$(a, a + 1, a + 1)$, which would imply that$n$is a divisor of$999$but not$1000$, or$(a, a, a + 1)$, which would imply that$n$is a divisor of$1000$but not$999$.$999 = 3^3 \cdot 37$has$4 \cdot 2 = 8$factors, and$1000 = 2^3 \cdot 5^3$has$4 \cdot 4 = 16$factors. However,$n = 1$does not work because$1$a divisor of both$999$and$1000$, and since$1$is counted twice, the answer is$16 + 8 - 2 = \boxed{\textbf{(A) }22}$.

Solution 2

Let $a = \left\lfloor \frac{998}n \right\rfloor$. Notice that if $\frac{998}n$ is divisible by $3$, then the three terms in the expression must be $(a, a, a)$, if $\frac{998}n$ is divisible by $3$, then the three terms in the expression must be $(a, a + 1, a + 1)$, and if if $\frac{1000}n$ is divisible by $3$, then the three terms in the expression must be $(a, a, a + 1)$.


Video Solution

https://youtu.be/Ozp3k2464u4

~IceMatrix

See Also

2020 AMC 10A (ProblemsAnswer KeyResources)
Preceded by
Problem 21
Followed by
Problem 23
1 2 3 4 5 6 7 8 9 10 11 12 13 14 15 16 17 18 19 20 21 22 23 24 25
All AMC 10 Problems and Solutions

The problems on this page are copyrighted by the Mathematical Association of America's American Mathematics Competitions. AMC logo.png